Area of a triangle using vectors

Click For Summary
To find the area of triangle PQR with vertices P = (1,1,1), Q = (0,3,1), and R = (0,1,4), the formula used is the magnitude of the cross product of vectors PQ and PR divided by two. The user initially calculated the cross product and arrived at an area of √13/2 but faced uncertainty about the correctness of their answer. Other participants pointed out potential errors in the cross product evaluation, specifically regarding the vector components. The discussion emphasizes the importance of correctly calculating the cross product to determine the accurate area of the triangle. Clarification on vector components is crucial for arriving at the correct solution.
Calpalned
Messages
297
Reaction score
6
## 1. Homework Statement
Let P = (1,1,1), Q = (0, 3, 1) and R = (0, 1, 4). Find the area of triangle PQR

Homework Equations


## \frac {|PQ × PR|}{2} ## = area (The crossproduct divided by two)

The Attempt at a Solution


I lost my answer key, so I want to check if my final answer of ## \frac {\sqrt {13}}{2} ## is right. Thanks everyone. If it isn't, I'll put up my work. ##
 
Physics news on Phys.org
Hey there Calpalned. I have worked the problem through, and I am not sure that your answer is correct. Are you sure you have evaluated the cross product correctly?
 
PhysyCola said:
Hey there Calpalned. I have worked the problem through, and I am not sure that your answer is correct. Are you sure you have evaluated the cross product correctly?

I took the cross product of ## <-1, 2, 0> ## and ## <-1, 0,3> ## and I got ## (0-0)-(-3-0)+(0--2) ## = ## <0, 3, 2> ## Taking the magnitude, I get the answer in my first post.
 
Calpalned said:
I took the cross product of ## <-1, 2, 0> ## and ## <-1, 0,3> ## and I got ## (0-0)-(-3-0)+(0--2) ## = ## <0, 3, 2> ## Taking the magnitude, I get the answer in my first post.
Not correct.

This should have vector components: ## (0-0)-(-3-0)+(0--2) ## . What you have is a scalar.

The x-component of the result, ##\ <0,\, 3,\, 2>\ ## is incorrect.
 
Question: A clock's minute hand has length 4 and its hour hand has length 3. What is the distance between the tips at the moment when it is increasing most rapidly?(Putnam Exam Question) Answer: Making assumption that both the hands moves at constant angular velocities, the answer is ## \sqrt{7} .## But don't you think this assumption is somewhat doubtful and wrong?

Similar threads

  • · Replies 2 ·
Replies
2
Views
2K
  • · Replies 7 ·
Replies
7
Views
1K
  • · Replies 2 ·
Replies
2
Views
1K
  • · Replies 2 ·
Replies
2
Views
2K
  • · Replies 3 ·
Replies
3
Views
3K
Replies
6
Views
8K
  • · Replies 12 ·
Replies
12
Views
2K
  • · Replies 30 ·
2
Replies
30
Views
4K
  • · Replies 6 ·
Replies
6
Views
2K
  • · Replies 5 ·
Replies
5
Views
2K